Finding the Angle of a Complex Number: Tips and Tricks

Click For Summary
To find the angle of a complex number represented by a transfer function, breaking it into real and imaginary parts is a common method, though users seek faster alternatives. A suggested approach involves multiplying by the conjugate of the denominator and using the arctangent function to calculate the phase angle. For specific cases where R and C are real, the phase can be directly determined using the formula arctan(wRC). However, the discussion emphasizes that users are looking for general rules or shortcuts for finding angles in complex numbers. Overall, while hints are provided, clarity on the reasoning behind them is requested.
asi123
Messages
254
Reaction score
0

Homework Statement



Hey guys.
I have the next transfer function

http://img195.imageshack.us/img195/7924/scan0002l.jpg

And I want to find the angle of it.
I know I can break it into REAL and IMAGINARY but I'm looking for a faster way, is there?

Thanks.

Homework Equations





The Attempt at a Solution

 
Last edited by a moderator:
Physics news on Phys.org
asi123 said:
I have the next transfer function

And I want to find the angle of it.

Hi asi123! :smile:

Hint: if w R and C are all real, then the angle (phase) of 1 + jwRC is arctan(wRC) :wink:
 
asi123 said:
I know I can break it into REAL and IMAGINARY but I'm looking for a faster way, is there?

Not really; finding the real and imaginary parts here is fairly quick. Just multiply both the numerator and denominator by the conjugate of the denominator, then \phi=\arctan\left(\frac{\text{Im}[H]}{\text{Re}[H]}\right).
 
gabbagabbahey said:
Not really; finding the real and imaginary parts here is fairly quick. Just multiply both the numerator and denominator by the conjugate of the denominator, then \phi=\arctan\left(\frac{\text{Im}[H]}{\text{Re}[H]}\right).

For this on, but I'm asking in general.

Thanks.
 
tiny-tim said:
Hi asi123! :smile:

Hint: if w R and C are all real, then the angle (phase) of 1 + jwRC is arctan(wRC) :wink:

Ok, I don't see any logic in that.
This is not a homework question, I know the answer, why would you give me a hint?
Is there a rule how to get to the angle?

Thanks.
 
Question: A clock's minute hand has length 4 and its hour hand has length 3. What is the distance between the tips at the moment when it is increasing most rapidly?(Putnam Exam Question) Answer: Making assumption that both the hands moves at constant angular velocities, the answer is ## \sqrt{7} .## But don't you think this assumption is somewhat doubtful and wrong?

Similar threads

  • · Replies 6 ·
Replies
6
Views
2K
  • · Replies 2 ·
Replies
2
Views
2K
  • · Replies 18 ·
Replies
18
Views
3K
  • · Replies 1 ·
Replies
1
Views
2K
  • · Replies 14 ·
Replies
14
Views
2K
  • · Replies 2 ·
Replies
2
Views
2K
  • · Replies 5 ·
Replies
5
Views
2K
Replies
2
Views
3K
  • · Replies 3 ·
Replies
3
Views
2K
Replies
8
Views
3K